LSAT and Law School Admissions Forum

Get expert LSAT preparation and law school admissions advice from PowerScore Test Preparation.

 brcibake
  • Posts: 55
  • Joined: Jul 19, 2017
|
#37453
I believe the first question about doctor's advice is a must be true question. Anyways, I do not understand why it should be C. I answered B but was stuck on this one. Thank you!
User avatar
 Jonathan Evans
PowerScore Staff
  • PowerScore Staff
  • Posts: 726
  • Joined: Jun 09, 2016
|
#37511
Hi, brcibake,

Let's take a look! It is indeed a Must Be True question, but as always, it's critical that we pay particular attention to the specifics or nuance of the question. Here the task is to find an answer that articulates a principle that most closely conforms to the circumstances outlined in the stimulus. In other words, our task is to analyze the stimulus, assess the statements, and notice connections or relationships between ideas in the statements.

In this case, we have one conditional statement that connects two ideas:
  • If a doctor gives a patient fewer choices rather than more choices, the patient is more likely to adhere to this doctor's advice.
In our analysis, we should note that we are seeking to articulate a principle or rule that describes the reasoning in the stimulus. Principles need to be abstract. In other words, we need to get outside the specifics of the problem. To create a good prediction/prephrase, let's drop the doctor/patient information and describe what's happening here.
  • If a someone is given fewer choices rather than more choices, this person is going to be more responsive to these choices.
This kind of description is accurate (it matches up with the scenario in the stimulus) but also broad enough to encompass a broader scope.

Match this prephrase up to the answer choices. Note that while (B) hits on some of the same ideas, it fails to be the most accurate match available. It introduces new concepts such as "the benefits that would result from each choice" and people's likes and dislikes. Given these problems, we should be careful with this answer; it will likely not be the best possible choice.

Now, match our prephrase up to (C). Note that we have to do a little bit of translation here. Let's start by matching up the concepts. This answer choice discusses the "tendency people have to alter their behavior" and " the number of alternatives available to them." Do these ideas sync up to "someone given fewer choices rather than more choices" and "responsive to these choices"? Yes:
  • "tendency people have to alter their behavior" ≈ "responsive to these choices"
  • "the number of alternatives available to them" ≈ "someone given fewer choices rather than more choices"
Thus, we have a conceptual match here. Next let's interpret this idea of inverse proportionality. Inverse proportionality means that as one quantity goes up, another quantity goes down. Is that what happens here? Yes. The more choices someone has, the less responsive this person is to those choices.

We have a complete match and have found the best answer! Good question! I hope this helps.

Get the most out of your LSAT Prep Plus subscription.

Analyze and track your performance with our Testing and Analytics Package.